Let $f$ be continuous on $[0,1]$ such that $int_0^x f = int_x^1 f$, what is $f$?












4












$begingroup$



Let $f$ be continuous on $[0,1]$, and suppose that for all $x$, $0<x<1$,



$int_0^x f = int_x^1 f$



Can you determine $f$?




I've argued the following:



Since $f$ is continuous on $[0,1]$, it follows that $f$ has an antiderivative $F$ on $I=[0,1]$.



Evaluating both sides of the equation we get



$F(x)-F(0) = F(1)-F(x)$=



$2F(x) = F(1) + F(0)$



$F(x) = frac{F(1) + F(0)}{2}$



Taking the derivative of both sides we get



$F'(x) = f(x) = frac{F'(1) + F'(0)}{2} = 0$ since $F(1) + F(0)$ is a constant.



I'm not sure if I'm going about this problem correctly.










share|cite|improve this question











$endgroup$








  • 3




    $begingroup$
    Aside from the "since $F(1) + F(0)$ missing a conclusion, yes this is correct. This is a ridiculously strong assumption: It says that no matter how you split the interval $[0, 1]$ into two subintervals, the mass that $f$ carries on each one is the same. That's too strong for any non-zero function.
    $endgroup$
    – T. Bongers
    Dec 1 '18 at 1:19
















4












$begingroup$



Let $f$ be continuous on $[0,1]$, and suppose that for all $x$, $0<x<1$,



$int_0^x f = int_x^1 f$



Can you determine $f$?




I've argued the following:



Since $f$ is continuous on $[0,1]$, it follows that $f$ has an antiderivative $F$ on $I=[0,1]$.



Evaluating both sides of the equation we get



$F(x)-F(0) = F(1)-F(x)$=



$2F(x) = F(1) + F(0)$



$F(x) = frac{F(1) + F(0)}{2}$



Taking the derivative of both sides we get



$F'(x) = f(x) = frac{F'(1) + F'(0)}{2} = 0$ since $F(1) + F(0)$ is a constant.



I'm not sure if I'm going about this problem correctly.










share|cite|improve this question











$endgroup$








  • 3




    $begingroup$
    Aside from the "since $F(1) + F(0)$ missing a conclusion, yes this is correct. This is a ridiculously strong assumption: It says that no matter how you split the interval $[0, 1]$ into two subintervals, the mass that $f$ carries on each one is the same. That's too strong for any non-zero function.
    $endgroup$
    – T. Bongers
    Dec 1 '18 at 1:19














4












4








4





$begingroup$



Let $f$ be continuous on $[0,1]$, and suppose that for all $x$, $0<x<1$,



$int_0^x f = int_x^1 f$



Can you determine $f$?




I've argued the following:



Since $f$ is continuous on $[0,1]$, it follows that $f$ has an antiderivative $F$ on $I=[0,1]$.



Evaluating both sides of the equation we get



$F(x)-F(0) = F(1)-F(x)$=



$2F(x) = F(1) + F(0)$



$F(x) = frac{F(1) + F(0)}{2}$



Taking the derivative of both sides we get



$F'(x) = f(x) = frac{F'(1) + F'(0)}{2} = 0$ since $F(1) + F(0)$ is a constant.



I'm not sure if I'm going about this problem correctly.










share|cite|improve this question











$endgroup$





Let $f$ be continuous on $[0,1]$, and suppose that for all $x$, $0<x<1$,



$int_0^x f = int_x^1 f$



Can you determine $f$?




I've argued the following:



Since $f$ is continuous on $[0,1]$, it follows that $f$ has an antiderivative $F$ on $I=[0,1]$.



Evaluating both sides of the equation we get



$F(x)-F(0) = F(1)-F(x)$=



$2F(x) = F(1) + F(0)$



$F(x) = frac{F(1) + F(0)}{2}$



Taking the derivative of both sides we get



$F'(x) = f(x) = frac{F'(1) + F'(0)}{2} = 0$ since $F(1) + F(0)$ is a constant.



I'm not sure if I'm going about this problem correctly.







calculus analysis






share|cite|improve this question















share|cite|improve this question













share|cite|improve this question




share|cite|improve this question








edited Dec 1 '18 at 1:21







K.M

















asked Dec 1 '18 at 1:17









K.MK.M

686412




686412








  • 3




    $begingroup$
    Aside from the "since $F(1) + F(0)$ missing a conclusion, yes this is correct. This is a ridiculously strong assumption: It says that no matter how you split the interval $[0, 1]$ into two subintervals, the mass that $f$ carries on each one is the same. That's too strong for any non-zero function.
    $endgroup$
    – T. Bongers
    Dec 1 '18 at 1:19














  • 3




    $begingroup$
    Aside from the "since $F(1) + F(0)$ missing a conclusion, yes this is correct. This is a ridiculously strong assumption: It says that no matter how you split the interval $[0, 1]$ into two subintervals, the mass that $f$ carries on each one is the same. That's too strong for any non-zero function.
    $endgroup$
    – T. Bongers
    Dec 1 '18 at 1:19








3




3




$begingroup$
Aside from the "since $F(1) + F(0)$ missing a conclusion, yes this is correct. This is a ridiculously strong assumption: It says that no matter how you split the interval $[0, 1]$ into two subintervals, the mass that $f$ carries on each one is the same. That's too strong for any non-zero function.
$endgroup$
– T. Bongers
Dec 1 '18 at 1:19




$begingroup$
Aside from the "since $F(1) + F(0)$ missing a conclusion, yes this is correct. This is a ridiculously strong assumption: It says that no matter how you split the interval $[0, 1]$ into two subintervals, the mass that $f$ carries on each one is the same. That's too strong for any non-zero function.
$endgroup$
– T. Bongers
Dec 1 '18 at 1:19










2 Answers
2






active

oldest

votes


















8












$begingroup$

Your method is generally correct, but I think this problem is easier than you think. We have:



$$int_0^x f(t)dt=int_x^1 f(t)dt=-int_1^x f(t)dt$$



Use the fundamental theorem of calculus to take the derivative of both sides:



$$f(x)=-f(x)rightarrow 2f(x)=0rightarrow f(x)=0$$



Thus, $f(x)=0$ for all $xin [0, 1]$.






share|cite|improve this answer









$endgroup$









  • 1




    $begingroup$
    Beat me to it by 2 minutes. +1
    $endgroup$
    – triple_sec
    Dec 1 '18 at 1:26



















7












$begingroup$

The use of the fundamental theorem of calculus makes the solution simpler. Since the two functions $xmapstoint_0^x f$ and $xmapstoint_x^1 f$ are identical on the interval $(0,1)$, it follows that their derivatives are also equal: $$f(x)=-f(x)quadtext{for every $xin(0,1)$.}$$ Hence, $f(x)=0$ for every $xin(0,1)$ and, by continuity, also for the endpoints $x=0$ and $x=1$.






share|cite|improve this answer









$endgroup$













    Your Answer





    StackExchange.ifUsing("editor", function () {
    return StackExchange.using("mathjaxEditing", function () {
    StackExchange.MarkdownEditor.creationCallbacks.add(function (editor, postfix) {
    StackExchange.mathjaxEditing.prepareWmdForMathJax(editor, postfix, [["$", "$"], ["\\(","\\)"]]);
    });
    });
    }, "mathjax-editing");

    StackExchange.ready(function() {
    var channelOptions = {
    tags: "".split(" "),
    id: "69"
    };
    initTagRenderer("".split(" "), "".split(" "), channelOptions);

    StackExchange.using("externalEditor", function() {
    // Have to fire editor after snippets, if snippets enabled
    if (StackExchange.settings.snippets.snippetsEnabled) {
    StackExchange.using("snippets", function() {
    createEditor();
    });
    }
    else {
    createEditor();
    }
    });

    function createEditor() {
    StackExchange.prepareEditor({
    heartbeatType: 'answer',
    autoActivateHeartbeat: false,
    convertImagesToLinks: true,
    noModals: true,
    showLowRepImageUploadWarning: true,
    reputationToPostImages: 10,
    bindNavPrevention: true,
    postfix: "",
    imageUploader: {
    brandingHtml: "Powered by u003ca class="icon-imgur-white" href="https://imgur.com/"u003eu003c/au003e",
    contentPolicyHtml: "User contributions licensed under u003ca href="https://creativecommons.org/licenses/by-sa/3.0/"u003ecc by-sa 3.0 with attribution requiredu003c/au003e u003ca href="https://stackoverflow.com/legal/content-policy"u003e(content policy)u003c/au003e",
    allowUrls: true
    },
    noCode: true, onDemand: true,
    discardSelector: ".discard-answer"
    ,immediatelyShowMarkdownHelp:true
    });


    }
    });














    draft saved

    draft discarded


















    StackExchange.ready(
    function () {
    StackExchange.openid.initPostLogin('.new-post-login', 'https%3a%2f%2fmath.stackexchange.com%2fquestions%2f3020884%2flet-f-be-continuous-on-0-1-such-that-int-0x-f-int-x1-f-what-is-f%23new-answer', 'question_page');
    }
    );

    Post as a guest















    Required, but never shown

























    2 Answers
    2






    active

    oldest

    votes








    2 Answers
    2






    active

    oldest

    votes









    active

    oldest

    votes






    active

    oldest

    votes









    8












    $begingroup$

    Your method is generally correct, but I think this problem is easier than you think. We have:



    $$int_0^x f(t)dt=int_x^1 f(t)dt=-int_1^x f(t)dt$$



    Use the fundamental theorem of calculus to take the derivative of both sides:



    $$f(x)=-f(x)rightarrow 2f(x)=0rightarrow f(x)=0$$



    Thus, $f(x)=0$ for all $xin [0, 1]$.






    share|cite|improve this answer









    $endgroup$









    • 1




      $begingroup$
      Beat me to it by 2 minutes. +1
      $endgroup$
      – triple_sec
      Dec 1 '18 at 1:26
















    8












    $begingroup$

    Your method is generally correct, but I think this problem is easier than you think. We have:



    $$int_0^x f(t)dt=int_x^1 f(t)dt=-int_1^x f(t)dt$$



    Use the fundamental theorem of calculus to take the derivative of both sides:



    $$f(x)=-f(x)rightarrow 2f(x)=0rightarrow f(x)=0$$



    Thus, $f(x)=0$ for all $xin [0, 1]$.






    share|cite|improve this answer









    $endgroup$









    • 1




      $begingroup$
      Beat me to it by 2 minutes. +1
      $endgroup$
      – triple_sec
      Dec 1 '18 at 1:26














    8












    8








    8





    $begingroup$

    Your method is generally correct, but I think this problem is easier than you think. We have:



    $$int_0^x f(t)dt=int_x^1 f(t)dt=-int_1^x f(t)dt$$



    Use the fundamental theorem of calculus to take the derivative of both sides:



    $$f(x)=-f(x)rightarrow 2f(x)=0rightarrow f(x)=0$$



    Thus, $f(x)=0$ for all $xin [0, 1]$.






    share|cite|improve this answer









    $endgroup$



    Your method is generally correct, but I think this problem is easier than you think. We have:



    $$int_0^x f(t)dt=int_x^1 f(t)dt=-int_1^x f(t)dt$$



    Use the fundamental theorem of calculus to take the derivative of both sides:



    $$f(x)=-f(x)rightarrow 2f(x)=0rightarrow f(x)=0$$



    Thus, $f(x)=0$ for all $xin [0, 1]$.







    share|cite|improve this answer












    share|cite|improve this answer



    share|cite|improve this answer










    answered Dec 1 '18 at 1:23









    Noble MushtakNoble Mushtak

    15.2k1735




    15.2k1735








    • 1




      $begingroup$
      Beat me to it by 2 minutes. +1
      $endgroup$
      – triple_sec
      Dec 1 '18 at 1:26














    • 1




      $begingroup$
      Beat me to it by 2 minutes. +1
      $endgroup$
      – triple_sec
      Dec 1 '18 at 1:26








    1




    1




    $begingroup$
    Beat me to it by 2 minutes. +1
    $endgroup$
    – triple_sec
    Dec 1 '18 at 1:26




    $begingroup$
    Beat me to it by 2 minutes. +1
    $endgroup$
    – triple_sec
    Dec 1 '18 at 1:26











    7












    $begingroup$

    The use of the fundamental theorem of calculus makes the solution simpler. Since the two functions $xmapstoint_0^x f$ and $xmapstoint_x^1 f$ are identical on the interval $(0,1)$, it follows that their derivatives are also equal: $$f(x)=-f(x)quadtext{for every $xin(0,1)$.}$$ Hence, $f(x)=0$ for every $xin(0,1)$ and, by continuity, also for the endpoints $x=0$ and $x=1$.






    share|cite|improve this answer









    $endgroup$


















      7












      $begingroup$

      The use of the fundamental theorem of calculus makes the solution simpler. Since the two functions $xmapstoint_0^x f$ and $xmapstoint_x^1 f$ are identical on the interval $(0,1)$, it follows that their derivatives are also equal: $$f(x)=-f(x)quadtext{for every $xin(0,1)$.}$$ Hence, $f(x)=0$ for every $xin(0,1)$ and, by continuity, also for the endpoints $x=0$ and $x=1$.






      share|cite|improve this answer









      $endgroup$
















        7












        7








        7





        $begingroup$

        The use of the fundamental theorem of calculus makes the solution simpler. Since the two functions $xmapstoint_0^x f$ and $xmapstoint_x^1 f$ are identical on the interval $(0,1)$, it follows that their derivatives are also equal: $$f(x)=-f(x)quadtext{for every $xin(0,1)$.}$$ Hence, $f(x)=0$ for every $xin(0,1)$ and, by continuity, also for the endpoints $x=0$ and $x=1$.






        share|cite|improve this answer









        $endgroup$



        The use of the fundamental theorem of calculus makes the solution simpler. Since the two functions $xmapstoint_0^x f$ and $xmapstoint_x^1 f$ are identical on the interval $(0,1)$, it follows that their derivatives are also equal: $$f(x)=-f(x)quadtext{for every $xin(0,1)$.}$$ Hence, $f(x)=0$ for every $xin(0,1)$ and, by continuity, also for the endpoints $x=0$ and $x=1$.







        share|cite|improve this answer












        share|cite|improve this answer



        share|cite|improve this answer










        answered Dec 1 '18 at 1:26









        triple_sectriple_sec

        15.8k21851




        15.8k21851






























            draft saved

            draft discarded




















































            Thanks for contributing an answer to Mathematics Stack Exchange!


            • Please be sure to answer the question. Provide details and share your research!

            But avoid



            • Asking for help, clarification, or responding to other answers.

            • Making statements based on opinion; back them up with references or personal experience.


            Use MathJax to format equations. MathJax reference.


            To learn more, see our tips on writing great answers.




            draft saved


            draft discarded














            StackExchange.ready(
            function () {
            StackExchange.openid.initPostLogin('.new-post-login', 'https%3a%2f%2fmath.stackexchange.com%2fquestions%2f3020884%2flet-f-be-continuous-on-0-1-such-that-int-0x-f-int-x1-f-what-is-f%23new-answer', 'question_page');
            }
            );

            Post as a guest















            Required, but never shown





















































            Required, but never shown














            Required, but never shown












            Required, but never shown







            Required, but never shown

































            Required, but never shown














            Required, but never shown












            Required, but never shown







            Required, but never shown







            Popular posts from this blog

            Plaza Victoria

            In PowerPoint, is there a keyboard shortcut for bulleted / numbered list?

            How to put 3 figures in Latex with 2 figures side by side and 1 below these side by side images but in...